You are on page 1of 10

Nanyang Technological University

School of Physical and Mathematical Sciences


Division of Mathematical Sciences

MH1811 Mathematics 2 Tutorial 3

√ 
x
1. Find the gradient vector ∇f of f (x, y) = tan−1 at P0 (4, 2) and sketch the level curve
y
that passes through P0 , the direction of the gradient vector, and the tangent line to the level
curve at P0 .
√ 
−1 x
[Solution] From f (x, y) = tan , we have
y
 
1 1 y
fx (x, y) = √ · √ = √ 2
x 2 2y x 2 x(y + x)
1+( )
y
 √  √
1 − x − x
and fy (x, y) = √ · = 2 .
x 2 y2 y +x
1+( )
y
Thus, we have  
1 1 1
∇f (P0 ) = (fx (P0 ), fy (P0 )) = ,− = (1, −4) .
16 4 16
√  √ !
−1 x 4
The equation of the level curve passing through P0 is tan = tan−1 = tan−1 1,
y 2

x √
which is = 1, i.e., y = x.
y

.............................................................................................

1
2. Verify that P (0, 1, 2) lies on the surface defined by
cos(πx) − x2 y + exz + yz = 4,
and find equations of the tangent plane and the normal line at P .
[Solution] The surface is the level set F (x, y, z) = 4 where
F (x, y, z) = cos(πx) − x2 y + exz + yz.
At the point P (0, 1, 2), we have
F (0, 1, 2) = cos(0) − 0 + e0 + (1)(2) = 1 + 1 + 2 = 4.
Thus, the point P (0, 1, 2) lies on the given surface.

The gradient vector ∇F (P ), which is normal to the given surface is computed as follows
∇F (x, y, z) = −π sin(πx) − 2xy + zexz , −x2 + z, xexz + y , so that ∇F (P ) = (2, 2, 1) .


The equation of the tangent plane at P (0, 1, 2) is ∇F (P ) · (x − 0, y − 1, z − 2) = 0, which is


2x + 2y + z = 2(0) + 2(1) + (2) = 4, i.e., 2x + 2y + z = 4.
The equation of the normal line at P (0, 1, 2) is
x = 2t, y = 1 + 2t, z = 2 + t.
.............................................................................................
3. Consider the sphere S : x2 + y 2 + z 2 − 8x − 6y − 8z + 24 = 0.

(a) Find the center and the radius of the sphere S.


[Solution] We do a completing squares for terms involving x, y and z:
x2 + y 2 + z 2 − 8x − 6y − 8z + 24 = 0 ⇐⇒ (x2 − 8x) + (y 2 − 6y) + (z 2 − 8z) + 24 = 0
⇐⇒ (x − 4)2 − 42 + (y − 3)2 − 32 + (z − 4)2 − 42 + 24 = 0
  

⇐⇒ (x − 4)2 + (y − 3)2 + (z − 4)2 = 17



The center of the sphere is (4, 3, 4) and its radius is 17.

(b) Show that sphere S and the ellipsoid E : 3x2 + 2y 2 + z 2 = 9 are tangent to each other at
the point P (1, 1, 2).
[Solution] We shall show that the normal vectors of the tangent planes to the sphere S
and ellipsoid E at the point P (1, 1, 2) are parallel.
Let F (x, y, z) = x2 + y 2 + z 2 − 8x − 6y − 8z + 24 and G(x, y, z) = 3x2 + 2y 2 + z 2 .
Note that F (1, 1, 2) = 0 and G(1, 1, 2) = 9. Thus the point P (1, 1, 2) lies on both sphere
S and ellipsoid E.
The tangent plane to S has normal vector ∇F = (2x − 8, 2y − 6, 2z − 8). The tangent
plane to E has normal vector ∇G = (6x, 4y, 2z).
At P (1, 1, 2), ∇F (P ) = (−6, −4, −4) and ∇G(P ) = (6, 4, 4), which are parallel but oppo-
site.
Therefore, both S and E have the same tangent plane at P (1, 1, 2). They are tangent to
each other at the point (1, 1, 2).

.............................................................................................

2

4. Find an equation for the plane that is tangent to the surface z = y − x at (1, 2, 1).
√ √
[Solution] Note that z = y − x is the graph of z = f (x, y) where f (x, y) = y − x. Therefore,
the equation of tangent plane at P is given by

z = f (1, 2) + ∇f (1, 2) · (x − 1, y − 2) .
 
√−1 , √1 −1 −1

We have f (1, 2) = 1 and ∇f (x, y) = 2 y−x 2 y−x
so that ∇f (1, 2) = 2 , 2 .
Thus, the required equation is z = f (1, 2) + ∇f (1, 2) · (x − 1, y − 2), which is
 
−1 1
z =1+ , · (x − 1, y − 2) ,
2 2

i.e., x − y + 2z = 1.

[Alternative Solution] Let F (x, y, z) = y − x − z. The surface is described by F (x, y, z) = 0
   
−1 1 −1 1
Note that ∇F (x, y, z) = √ , √ , −1 and ∇F (1, 2, 1) = , , −1 .
2 y−x 2 y−x 2 2

An equation for the tangent to the surface z = y − x at (1, 2, 1) is
 
−1 1
, , −1 · (x − 1, y − 2, z − 1) = 0,
2 2
which gives
−1 1 −1 1
x+ y−z = (1) + (2) + (−1)(1) = −1/2,
2 2 2 2
i.e., x − y + 2z = 1.

.............................................................................................
p
5. Find the linear approximation of the function f (x, y) = 20 − x2 − 7y 2 at (2, 1) and use it to
approximate f (1.95, 1.08). (Answer: 2.8467)
−1 −1
[Solution] Note that ∇f (x, y) = p (x, 7y). and ∇f (2, 1) = (2, 7).
2
20 − x − 7y 2 3
Thus, we have
−1
L(x, y) = f (2, 1) + ∇f (2, 1) · (x − 2, y − 1) = 3 + (2, 7) · (x − 2, y − 1) .
3
To approximate f (1.95, 1.08) by L(1.95, 1.08), we have
1
f (1.95, 1.08) ≈ L(1.95, 1.08) = 3 + (−2, −7) · (1.95 − 2, 1.08 − 1)
3
1 0.46
=3+ (−2(−0.05) − 7(0.08)) = 3 − ≈ 2.8467.
3 3
.............................................................................................

3
1
6. Use linear approximation to estimate .
3.952+ 3.12
1 1
[Solution] Let f (x, y) = 2 so that = f (3.95, 3.1) and (a, b) = (4, 3).
x + y2 3.952 + 3.12
 
−2x −2y
Then ∇f (x, y) = (fx , fy ) = , .
(x2 + y 2 )2 (x2 + y 2 )2
1
The value = f (3.95, 3.1) is approximated as follows:
3.95 + 3.12
2

f (3.95, 3.1) ≈ L(3.95, 3.1) = f (4, 3) + fx (4, 3) (3.95 − 4) + fy (4, 3) (3.1 − 3)

1 −2(4) −2(3)
= + (−0.05) + (0.1)
25 252 252
25 + 0.4 − 0.6
= = 0.03968
252
.............................................................................................

7. Determine the “total differential” (dz) and the “increment” (∆z) of the following functions
Recall dz = fx (a, b) · (dx) + fy (a, b) · (dy)
y
(a) z = at x = 2, y = 1, dx = 0.1, dy = 0.2.
x
∂z −y ∂z 1 ∂z −1 ∂z 1
[Solution] = 2, = so that | = , | = . Therefore, we have
∂x x ∂y x ∂x (2,1) 4 ∂y (2,1) 2
−1 1
dz = · (0.1) + · (0.2) = −0.025 + 0.1 = 0.075
4 2
1.2 1
Note that ∆z = − ≈ 0.071429.
2.1 2
xy
(b) z = at x = 2, y = 1, dx = 0.01, dy = −0.03.
x2+ y2
∂z y(x2 + y 2 ) − (xy)(2x) y(y 2 − x2 ) ∂z x(x2 − y 2 )
[Solution] = 2 2 2
= 2 2 2
, = 2
∂x (x + y ) (x + y ) ∂y (x + y 2 )2
Therefore, we have
−3 6
dz = · (0.01) + · (−0.03) = −0.0084.
25 25
(2.01)(0.97) 2
Note that ∆z = − ≈ −0.00857.
((2.01)2 + (0.97)2 ) 5
.............................................................................................

4
8. Consider a closed rectangular box with a square base of side 3 unit, and height 5 unit. If the
side is measured with an error at most 0.02 unit and the height is measured with an error
at most 0.01 unit, use differentials to estimate the maximum possible error in computing the
volume of the box.
[Solution] Let x be the length of the side of the square and h be the height of the box.
Then the volume V of the box is V (x, h) = x2 h.
We have Vx = 2xh and Vh = x2 .

At x = 3, and h = 5, the volume is V (3, 5) = 45 and Vx = 30 and Vh = 9.

The side is measured with an error at most 0.02 unit means that |dx| ≤ 0.02, while the height
is measured with an error at most 0.01 unit means |dh| ≤ 0.01.
The maximum possible error can be estimated by dV as follows:

|dV | = |Vx (3, 5) · (dx) + Vh (3, 5) · (dh)| = |30 · (dx) + 9 · (dh)|

≤ 30|dx| + 9|dh| ≤ 30(0.02) + 9(0.01) = 0.69


|{z}

Remark (*) We use the triangle inequality |a + b| ≤ |a| + |b| and the property |ab| = |a| · |b|.

dV 0.69
The maximum relative error is ≤ .
V 45
dV 0.69
The maximum relative error in percentage is (100%) ≤ (100%).
V 45

.............................................................................................

5
9. Let f (s, t) = s2 et . Find the directional derivative of f at the point (2, 0) in the direction of
the vector v = i + j.
1
[Solution] Recall that Du f (P0 ) = ∇f (P0 ) · u, where u = v, the unit vector along v.
kvk

√ √ 1 1
We have kvk = 12 + 12 = 2 so that that u = √ (i + j) = √ (1, 1), and
2 2

∇f (P0 ) = (fs (P0 ), ft (P0 )) = 2set , s2 et |P0 = (4, 4) .




Thus, the required directional derivative is


1 8 √
Du f (P0 ) = (4, 4) · √ (1, 1) = √ = 4 2.
2 2
.............................................................................................

10. Suppose f (x, y) = sin (xy).


−→
(a) What is the rate of change of in f at P (1, 0) in the direction P R where R is the point
R (2, 3)? Is it increasing or decreasing?
(b) Find the maximum rate of change of f at P (1, 0) and the direction in which it occurs.
(c) Find the minimum rate of change of f at Q (π, 1) and the direction in which it occurs.

[Solution] We have ∇f (x, y) = (y cos(xy), x cos(xy)).


−→
(a) Note the the direction v = P R = (1, 2).
The required rate of change is Du f (P ) = ∇f (P ) · u where u is the unit vector along v.
1 1 1
We have u = v= √ (1, 3) = √ (1, 3) and ∇f (1, 0) = (0, 1).
kvk 2
1 +3 2 10
Thus, the required rate is
1 1
Du f (P ) = (0, 1) · √ (1, 3) = √ (3) .
10 10
The function f is increasing at P (1, 0) in the direction v = (1, 3).
(b) We have ∇f (1, 0) = (0, 1) and k∇f (1, 0)k = 1.
The maximum rate of change of f (x, y) at P (1, 0) occurs along the direction
1
u= ∇f (1, 0) = (0, 1), and its rate is k∇f (1, 0)k = 1.
k∇f (1, 0)k
(c) We have ∇f (π, 1) = (−1, −π).
The minimum rate of change of at Q (π, 1) is
p
−k∇f (π, 1)k = − 1 + π 2 ,

−∇f (π, 1)
and it occurs along the direction = √ 1 (1, π).
k∇f (π, 1)k 1+π 2

.............................................................................................

6
 
x−y 1 3
11. Let f (x, y) = . Find the direction u and the values of Du f − , for which
x+y 2 2
   
1 3 1 3
(a) Du f − , = −1. Indicate the vector u and ∇f − , on the same xy-plane.
2 2 2 2
 
1 3
(b) Du f − , = 0.
2 2
 
1 3
[Solution] Our aim is to find unit vectors u = (u1 , u2 ) such that Du f − , equals to the
2 2
stated value.
   
1 3 1 3
Note that Du f − , = u · ∇f − , , where u21 + u22 = 1.
2 2 2 2
 
1 3
We find ∇f − , as follows
2 2
(x + y) − (x − y) 2y −(x + y) − (x − y) −2x
fx = 2
= 2
, fy = 2
= .
(x + y) (x + y) (x + y) (x + y)2
 
1 3
Thus, we have ∇f − , = (3, 1).
2 2

(a) We shall find unit vectors u = (u1 , u2 ) such that u21 + u22 = 1 and
   
1 3 1 3
Du f − , = u · ∇f − , = −1, i.e., u · (3, 1) = −1, i.e., 3u1 + u2 = −1.
2 2 2 2
We solve u21 + u22 = 1 and 3u1 + u2 = −1.
Substituting u2 = −1 − 3u1 into u21 + u22 = 1:
2
u21 + −1 − 3u21 = 1, giving (10u1 + 6)u1 = 0.

3 4
Thus, we have u1 = 0 ( and u2 = −1) or u1 = − (and u2 = ).
  5 5
3 4
Therefore, u = (0, −1) or u = − , .
5 5
   
1 3 1 3
(b) Similarly, we solve Du f − , = 0, i.e., u · ∇f − , = 0, i.e., 3u1 + u2 = 0. Thus,
2 2 2 2
u2 = −3u1 .
For u to be a unit vector, we must have u21 + u22 = 1.
1 −3 −1
Thus we have 10u21 = 1, which gives u1 = √ (and u2 = √ ) or u1 = √ (and
10 10 10
3
u2 = √ )
10
We have
±1
u = √ (1, −3) .
10
.............................................................................................

7
12. The temperature T in a metal ball is inversely proportional to the distance from the center of
the ball which we take to be the origin. The temperature at the point (1, 2, 2) is 120◦ C.

(a) Find the rate of change of T at (1, 2, 2) in the direction toward the point (2, 1, 3).
(b) Show that at any point in the ball the direction of greatest rate of increase in temperature
is given by a vector that points toward the origin.

1 k
(A variable a is said to be inversely proportional to another variable b, means a ∝ , i.e., a =
b b
for some constant k.)
p
[Solution] The distance from a point (x, , y, z) to the center is x2 + y 2 + z 2 .
From the information provided, we have
K
T (x, y, z) = p , where K is a constant.
x + y2 + z2
2

K K
Since T (1, 2, 2) = 120, we have 120 = √ = , which gives K = 360.
12 2
+2 +2 2 3
360
Therefore, we have T (x, y, z) = p , which gives
x2 + y 2 + z 2

−360
∇T (x, y, z) = p (x, y, z) .
(x2 + y 2 + z 2 )3

(a) The rate of change of T at (1, 2, 2) in the direction toward the point (2, 1, 3) is Du T (1, 2, 2)
where u is the unit vector along the direction from (1, 2, 2) to (2, 1, 3). Thus, we have
1
u = √ (1, −1, 1).
3
−360 1 −40
Du f (1, 2, 2) = ∇T (1, 2, 2) · u = 3
(1, 2, 2) · √ (1, −1, 1) = √ .
3 3 3 3
−360
(b) The gradient vector ∇T (x, y, z) = p (x, y, z), which provides the direction
(x2 + y 2 + z 2 )3
−360
of greatest rate of increase in temperature. Since p < 0, the vectors
(x2 + y 2 + z 2 )3
−−→
∇T (x, y, z) and OP = (x, y, z) are opposite to each other.
Therefore, we conclude that at any point in the ball the direction of greatest increase in
temperature is given by a vector that points toward the origin.

.............................................................................................

8
13. Let f be a function of two variables that has continuous partial derivatives and consider the
points A(1, 3), B(3, 3), C(1, 7) and D(6, 15). The directional directive of f at A in the direction
−−→ −→
of the vector AB is 3 and the directional directive of f at A in the direction of the vector AC
−−→
is 26. Find the directional derivative of f at A in the direction of the vector AD.

[Solution] Note that


−−→ −−→ −→ −−→
AB = OB − OA = (2, 0) so that the unit vector along AB is i,
−→ −−→ −→ −→
AC = OC − OA = (0, 4) so that the unit vector along AC is j,
and
−−→ −−→ −→ −−→ 1
AD = OD − OA = (5, 12) so that the unit vector along AD is u = (5, 12).
13
We have Di f (1, 3) = ∇f (1, 3) · i = (fx (1, 3), fy (1, 3)) · (1, 0) = fx (1, 3),
and, similarly,
Dj f (1, 3) = ∇f (1, 3) · j = fy (1, 3).

Given that Di f (1, 3) = 3 and Dj f (1, 3) = 26, we have fx (1, 3) = 3 and fy (1, 3) = 26.
−−→
Therefore, we have the directional derivative of f at A in the direction of the vector AD is

1 15 + 12(26) 327
Du f (13) = (3, 26) · (5, 12) = = .
13 13 13
.............................................................................................

14. Find all points at which the direction of maximum rate of change of the function f (x, y) =
x2 + y 2 − 2x − 4y is i + j. (Answer : All points on the line y = x + 1, where x > 1.)
[Solution] Note that ∇f (x, y) = (2x − 2, 2y − 4).
The direction of maximum rate of change of the function is along ∇f . Thus we want to have
∇f (x, y) = λ(i + j), where λ > 0.
This gives 2x − 2 = λ and 2y − 4 = λ.
Eliminating λ, we obtain y = x + 1, and x > 1.
Thus points (x, y) at which the direction of maximum rate of change of the function f (x, y) =
x2 + y 2 − 2x − 4y is i + j are all points on the line y = x + 1, where x > 1.
.............................................................................................

9
15. John is climbing a hill whose shape is given by the equation z = 1000 − 0.01x2 − 0.02y 2 , where
x, y and z are measured in meters. John is standing at a point with coordinates (50, 80, 847).
The positive x-axis points east and the positive y-axis points north.

(a) If John walks northwest, will he start to ascend or descend? At what rate?
(b) In which direction is the slope largest? What is the rate of ascent in that direction?

[Solution] ∇z(x, y) = (−0.02x, −0.04y) and ∇z(50, 80) = (−1, −3.2)


−1 √1 j.
(a) Along northwest, we take u = √
2
i+ 2
Thus, the required rate of change is
 
−1 1 −2.2
Du z(50, 80) = ∇z(50, 80) · u = (−1, −3.2) · √ ,√ = √ ≈ −1.5556.
2 2 2
It is the rate of descend.
(b) The slope is largest along ∇z(50, 80) = (−1, −3.2) and the corresponding unit vector is
1
u= p (−1, −3.2) ≈ (−0.298, −0.954) .
(−1) + (−3.2)2
2

p
The rate of ascent in this direction is k∇z(50, 80)k = (−1)2 + (−3.2)2 ≈ 3.35261.

.............................................................................................

16. Let f (x, y) = 3 xy.

(i) Use the definition of partial derivatives to find fx (0, 0) and fy (0, 0).
(ii) Use the definition of directional derivative to show that Du f (0, 0) does not exist in other
direction u = (a, b) where ab 6= 0.

[Solution] Note that f (0, 0) = 0, f (h, 0) = 0 and f (0, h) = 0.

(i) By definition, we have


f (h, 0) − f (0, 0) 0−0
fx (0, 0) = lim = lim = 0.
h→0 h h→0 h
Similarly, we have
f (0, h) − f (0, 0) 0−0
fy (0, 0) = lim = lim = 0.
h→0 h h→0 h
(ii) By the definition of directional derivative, we have

h2 ab √
3
f (ha, hb) − f (0, 0) 3 1
Du f (0, 0) = lim = lim = ab lim √
3
,
h→0 h h→0 h h→0 h2
which does not exist. Thus, Du f (0, 0) does not exist in all other direction u = (a, b) where
ab 6= 0.

.............................................................................................

10

You might also like